Una Propiedad Exponencial

Sea f:\mathbb{R}\rightarrow\mathbb{R} una función continua tal que

f(x+y)=f(x)f(y) para todos x,y\in\mathbb{R}

entonces f\equiv 0 o f(x)=a^{x} para algún a\in\mathbb{R}^{+}.

En efecto, lo primero que debemos notar es que o bien f(0)=0 o bien f(0)=1, para mostrarlo es suficiente escribir f(0)=f(0+0)=f(0)^{2} lo que implica el resultado. Si f(0)=0 entonces f(x)=f(x+0)=f(x)f(0)=0 para cada x\in\mathbb{R}. Si no, entonces f(0)=1 y f(1)=f(\frac{1}{n}\cdot n)=f(\frac{1}{n})^{n}, lo que implica que f(1)>0 pues para n suficientemente grande, f(1/n)>0 por continuidad en el origen. Es fácil ver también que lo último implica que f(x)>0 para cada x\in\mathbb{R}.

Observemos ahora que en cada conjunto compacto (cerrado y acotado) la función es Riemann integrable por continuidad, por lo tanto

\displaystyle{\int_{0}^{x}f(t)dt=\lim_{n\rightarrow\infty}\sum_{k=0}^{n-1}f\left(\frac{kx}{n}\right)\frac{x}{n}}

Por otro lado, para todo n natural y y\in\mathbb{R}, f(ny)=f(y)^{n} y f(y/n)=f(y)^{1/n}, de donde

\displaystyle{f\left(\frac{kx}{n}\right)=f(x)^{\frac{k}{n}}}

Lo que implica

\displaystyle{\int_{0}^{x}f(t)dt=\lim_{n\rightarrow\infty}\frac{x}{n}\sum_{k=0}^{n-1}f(x)^{\frac{k}{n}}}

y luego la suma geométrica nos entrega

\displaystyle{\int_{0}^{x}f(t)dt=\lim_{n\rightarrow\infty}\frac{x}{n}\frac{f(x)-1}{f(x)^{1/n}-1}}.

Recordemos ahora que \displaystyle{\lim_{n\rightarrow\infty}n(y^{1/n}-1)} para cada y\in\mathbb{R}^{+}, entonces

\int_{0}^{x}f(t)dt=\dfrac{x(f(x)-1)}{\log{f(x)}}.

En el caso particular de $x=n$ obtenemos $f(n)=f(1)^{n}$ y por lo tanto

\int_{0}^{n}f(t)dt=\frac{f(n)-1}{\log{f(1)}}.

Por otro lado un simple cambio de variables implica

\int_{x}^{x+n}f(t)dt=\int_{0}^{n}f(t+x)dt=f(x)\int_{0}^{n}f(t)dt

por lo que si derivamos respecto a x, nos queda

f(x+n)-f(x)=\frac{df}{dx}(x)\frac{f(n)-1}{\log{f(1)}},

pero  f(x+n)-f(x)=f(x)(f(n)-1), entonces

f(x)\log{f(1)}=\frac{df}{dx}(x).

Basta considerar a=f(1) y resolver la ecuación diferencial para concluir que f(x)=a^{x}.

Otra forma de resolver el problema es una perspectiva de la densidad. En efecto, si dos funciones f,g:\mathbb{R}\rightarrow\mathbb{R} continuas coinciden en un conjunto denso, entonces las funciones coinciden en toda la recta real. Para ello basta considerar un punto arbitrario y estudiar la convergencia a tal punto por sucesiones del conjunto denso, los límites deben ser iguales al evaluar en cada función. Finalmente, para cada p,q\in\mathbb{Q}, con q\neq 0, entonces f(p/q)=f(1)^{p/q}. Es decir, para a=f(1), las funciones f(x) y g(x)=a^{x} coinciden sobre \mathbb{Q}. Nuestra primera observación en este párrafo concluye el resultado.

Sobre cardinalidades

Consideremos un conjunto no vacío X y el conjunto de las partes de X (o dicho de otra forma, el conjunto de subconjuntos de X), denotado por \mathcal{P}(X). Entonces X\mathcal{P}(X) tienen cardinalidades distintas.

 

Recordemos que cuando los conjuntos son infinitos, decimos que dos conjuntos A,B tienen la misma cardinalidad si existe una biyección entre ellos; i.e. si existe \phi:A\rightarrow B biyectiva.

Supongamos entonces que X\mathcal{P}(X) tienen la misma cardinalidad y denotemos \phi su biyección.

Definimos entonces el siguiente conjunto

E:=\{x\in X\textmd{ }|\textmd{ }x\not\in\phi(x)\}

Esta definición tiene sentido por lo siguiente: \phi(x) denota un subconjunto de X por lo que es natural preguntarse si x está o no en su imagen.


Observación
: E\neq\emptyset pues en particular \emptyset\in\mathcal{P}(X) por lo que (por definición de biyección) debe existir algún elemento y\in X tal que \phi(y)=\emptyset, de donde se concluye que y\not\in\phi(y) y luego y\in E

Como E\subset X, y nuevamente por definición de biyección, se tiene que debe existir z\in X tal que \phi(z)=E. Entonces, ¿z está o no en E? si lo estuviera quiere decir (por definición de E) que z\not\in\phi(z)=E, lo cual sería absurdo. Si no lo estuviera entonces nuevamente por definición del conjunto z\in\phi(z)=E lo cual es también un absurdo. Hemos llegado entonces a una contradicción lógica, no puede existir ningún elemento en X que tenga por imagen E y por lo tanto no puede existir una biyección.

 

Una aplicación directa de este resultado es el hecho de que los naturales no se encuentran en biyección con las sucesiones a valores en \{0,1\}. Dicho mas precisamente \mathbb{N} y \{0,1\}^{\mathbb{N}} no tienen la misma cardinalidad. De hecho mostraremos algo más concreto: \mathcal{P}(\mathbb{N})\{0,1\}^{\mathbb{N}} tienen la misma cardinalidad (y por lo que mostramos anteriormente esto demuestra lo que queremos).

Definimos \phi:\mathcal{P}(\mathbb{N})\rightarrow\{0,1\}^{\mathbb{N}} como sigue: si E\in\mathcal{P}(\mathbb{N}) es decir E\subset X, escribimos \phi(E)=\{x_{i}\}_{i\in\mathbb{N}}, donde x_{i}=1 si i\in E y x_{i}=0 si i\not\in E.

Es claro por la definición que \phi es efectivamente una función, y además es biyectiva pues si \phi(E)=\phi(F) para E,F\subset X, entonces (para \phi(E)=\{x_{i}\}_{i\in\mathbb{N}} y \phi(F)=\{y_{j}\}_{j\in\mathbb{N}}) se tiene x_{i}=y_{i} para todo i\in\mathbb{N}, de donde i\in E\Leftrightarrow i\in F, lo que concluye que E=F. Finalmente una sucesión \{x_{i}\}_{i\in\mathbb{N}} define al conjunto A=\{i\in\mathbb{N}:x_{i}=1\} y por definicion \{x_{i}\}_{i\in\mathbb{N}}=\phi(A) (la construcción de A nos entrega realmente la inversa de \phi).


Observación
: Muchas veces en los textos se denota la cardinalidad de \mathcal{P}(\mathbb{N}) como 2^{\mathbb{N}}. Tiene bastante sentido si lo pensamos por el hecho de que \{0,1\} tiene dos elementos.

 

Como ultima observación, uno puede definir una función inyectiva X\rightarrow\mathcal{P}(\mathbb{X}) que asocia a cada elemento x\in X al conjunto singleton \{x\}\subset X. No deben confundir, x es solo un elemento del conjunto X y su singleton es el conjunto que contiene a un solo elemento de X, solo contiene a x. Esto nos dice que la cardinalidad de X es estrictamente menor que la cardinalidad de  \mathcal{P}(\mathbb{X}).

Sobre continuidad y continuidad uniforme

Recordemos que en \mathbb{R} los conjuntos compactos son aquellos cerrados y acotados. Estos tienen otra caracterización dada por el hecho de que toda sucesión en un tal conjunto contiene una subsucesión convergente (que converge en el conjunto).

Consideremos entonces I=[a,b]\subset\mathbb{R}, el cual es un conjunto compacto. En este post daremos una propiedad de las funciones continuas sobre este tipo de intervalos que trae consecuencias potentes en su estudio. En un futuro post daremos una de estas consecuencias en detalle.

Recuerdo: Una función continua f:U\subset\mathbb{R}\rightarrow\mathbb{R} se dice Uniformemente Continua si para cada \varepsilon>0 existe \delta>0 tal que para todos x,y\in U se tiene

|x-y|<\delta  \Rightarrow |f(x)-f(y)|<\varepsilon

Es sumamente importante darse cuenta de la diferencia con la continuidad. En estricto rigor la continuidad de una función nos dice que \delta existe y depende del punto (en el que evaluamos la continuidad) y de \varepsilon. Según nuestra definición de continuidad uniforme, tal \delta no depende del punto en cuestión. Podemos decir de manera poco formal que nuestro \delta sirve en cada punto, es uniforme en el conjunto U (¡de ahí el nombre de la continuidad!).

Un ejemplo de función continua y no uniformemente continua es f:(0,1)\rightarrow\mathbb{R} definida por f(x)=\frac{1}{x} (¡Compruébenlo!).

 

Proposición: Sea f:I\rightarrow\mathbb{R} continua. Entonces f es uniformemente continua.

Demostración:

El método de la prueba es por contradicción, es decir, supondremos que nuestra función es continua pero no uniformemente continua. En algún momento llegaremos a una contradicción con alguna hipótesis. El punto crucial en la demostración es la estructura topológica de nuestro dominio: I es un compacto.

En lenguaje matemático formal, la continuidad uniforme es equivalente a

\forall\varepsilon>0 \exists\delta>0 tales que \forall x,y\in I se tiene

|x-y|<\delta  \Rightarrow |f(x)-f(y)|<\varepsilon

si negamos toda esta oración, nos queda

\exists\varepsilon_{0}>0 tal que \forall\delta>0 existen x_{\delta},y_{\delta}\in I que satisfacen

|x_{\delta}-y_{\delta}|<\delta y |f(x_{\delta})-f(y_{\delta})|\geq\varepsilon_{0}.

En particular, para \delta_{n}=\frac{1}{n}, existen x_{n},y_{n}\in I con la propiedad anterior.

Ahora usaremos la compacidad, \{x_{n}\}_{n} es una sucesión en un conjunto compacto, por lo tanto contiene una subsucesión convergente. Por simplicidad de notación supondremos entonces, sin perder la generalidad, que \{x_{n}\}_{n} es convergente, digamos que converge a x_{0}\in I. Notar que

|x_{0}-y_{n}|\leq|x_{0}-x_{n}|+|x_{n}-y_{n}|<|x_{0}-x_{n}|+\delta_{n}

y ya que ambos términos a la derecha tienden a 0 se tiene que \{y_{n}\}_{n} es convergente y converge a x_{0}.

Por último, la función es continua, por lo que en particular es continua en x_{0}, es decir, para todo \varepsilon>0 existe \delta tal que |x-x_{0}|<\delta implica |f(x)-f(x_{0})|<\varepsilon. Escogemos \varepsilon=\frac{\varepsilon_{0}}{2} y luego existe un \delta_{0} con la propiedad de la definición de continuidad. Ya que tanto \{x_{n}\} como \{y_{n}\} convergen a x_{0} se tiene que desde n suficientemente grande,

x_{n}\in B(x_{0},\delta_{0}) y y_{n}\in B(x_{0},\delta_{0})

y luego (para n suficientemente grande)

|f(x_{n})-f(y_{n})|\leq|f(x_{n})-f(y_{0})|+|f(x_{0})-f(y_{n})|<\frac{\varepsilon_{0}}{2}+\frac{\varepsilon_{0}}{2}=\varepsilon_{0}

es decir

|f(x_{n})-f(y_{n})|<\varepsilon_{0}

pero nuestra hipótesis era justamente que para tales sucesiones |f(x_{n})-f(y_{n})|\geq\varepsilon_{0}. Hemos llegado a un absurdo.

Esto muestra que una función continua definida en un intervalo compacto debe ser uniformemente continua.

\blacksquare

Sobre subsucesiones II

Consideremos una sucesión de números reales \{x_{n}\}_{n\in\mathbb{N}}.

Definición: Una partición (finita) de \{x_{n}\}_{n\in\mathbb{N}} es una colección finita de subsucesiones tal que, como conjuntos, son mutuamente disjuntas y su unión es la sucesin original.

Proposición: Si todas las subsucesiones de la partición convergen a un mismo límite, entonces la sucesión original converge. Más aún su límite es el límite de cada subsucesión.

Demostración: Sean \phi_{1},...,\phi_{m}:\mathbb{N}\rightarrow\mathbb{N} funciones estrictamente crecientes que definen las subsucesiones de la partición. Es decir, \bigcup_{i=1}^{m}\{x_{\phi_{i}(n)}\}_{n\in\mathbb{N}}=\{x_{n}\}_{n\in\mathbb{N}} y \{x_{\phi_{i}(n)}\}_{n\in\mathbb{N}}\cap\{x_{\phi_{j}(n)}\}_{n\in\mathbb{N}}=\emptyset para todos i,j distintos. Por hipótesis, para cada i\in\{1,...,m\} se tiene

\lim_{n\rightarrow\infty}x_{\phi_{i}(n)}=L

por lo que para cada \varepsilon>0 existe N(i)\in\mathbb{N} tal que n>N(i) implica |x_{\phi_{i}(n)}-L|<\varepsilon.

Definimos N=\max\{\phi_{1}(N(1)),...,\phi_{m}(N(m))\}, entonces para cada n>N se tiene que x_{n} es el término de alguna subsucesión (de ahí la importancia que la unión de los términos de las subsucesiones sea la sucesión original) por lo que necesariamente |x_{n}-L|<\varepsilon. Esta es exactamente la definicion de límite, por lo que concluye la demostración.

\blacksquare

Observación: Podríamos debilitar un poco las hipótesis y seguir manteniendo el resultado. Por ejemplo, la condición de que las subsucesiones sean mutuamente disjuntas no se usa rigurosamente (la usamos para poder asegurar que la sucesión original está suficientemente cerca del límite sin ponernos en demasiados casos) pero en muchos casos es mas útil trabajar bajo estas condiciones. Por lo demás, la demostración resulta mucho mas sencilla.

Sobre subsucesiones

1.- Si una sucesión monótona tiene una subsucesión convergente, demuestre que la sucesión converge.

Solución: Probaremos que la sucesión está acotada, ya que si ésta es monótona y está acotada, converge.

Sea (x_k) una sucesión monótona (supongamos creciente). En particular, tenemos una subsucesión creciente, y por enunciado, convergente.

Definimos la subsucesión (x_{n_{k}}) como {x_{n_{1}}x_{n_{2}}, …, x_{n_{k}} , … }=X tal que x_{n_{k}}\leq x_{n_{k+1}}

Ahora, como sabemos que la subsucesión converge, ésta está acotada. Luego x_{n_{k}}\rightarrow L cuando k\rightarrow\infty, donde L=\sup{X}. Entonces x_{n_{k}}\leq L para todo k\in N.

Por otro lado, como x_k es creciente, existe f: \mathbb{N}\rightarrow \mathbb{N} tal que f(k)=n_k con f creciente, es decir que  k\leq n_k para todo k en \mathbb{N}, por lo tanto x_k \leq x_{n_k}\leq L, luego x_k\leq L

Entonces, como (x_k) está acotada y es monótona (creciente), converge.

Un problema clásico

Consideremos un cuadrado de lado 1. Sobre una de sus esquinas dibujamos un cuadrado de lado \frac{1}{2}; sobre el borde exterior de este nuevo cuadrado dibujamos en una de sus esquinas un cuadrado de lado \frac{1}{4}, y así de manera recursiva… sobre el borde exterior del cuadrado n-ésimo, en una de sus esquinas, dibujamos un cuadrado de lado \frac{1}{2^{n+1}}. Podemos hacer esto de manera ordenada como en la siguiente figura. La pregunta es… ¿cuál es la suma total de las áreas de los cuadrados así formados?

fig1

Naturalmente estamos sumando una cantidad infinita de cuadrados, por lo tanto, a priori, la suma de las áreas no tiene porque ser finita. Sin embargo, podemos utilizar el siguiente resultado

Proposición: Sea \{a_{n}\}_{n>0} una sucesión de números reales, no decreciente y acotada superiormente. Entonces el límite \lim_{n\rightarrow\infty}a_{n} existe (y es finito).

Demostración:

Sea A=\{a_{n}\}_{n>0} visto como subconjunto de los numeros reales. El hecho de que la sucesión esté acotada superiormente implica que el conjunto A tiene supremo; llamemoslo L. Debemos mostrar que L es el límite de la sucesión.

En efecto, si consideramos \varepsilon>0, entonces L-\varepsilon no es cota superior del conjunto, pues de serlo él sería menor que la menor cota superior del conjunto A, contradiciendo que L es supremo. Por lo tanto, existe un elemento a_{N} de la sucesión para el cual se tiene L-\varepsilon<a_{N}\leq L. Por otro lado la sucesión es no decreciente, por lo que para cada natural n\geq N se tiene a_{N}\geq a_{n} . Así mismo, como A está acotado superiormente por el supremo, se satisface a_{n}\leq L (en particular) para cada n\geq N. Hemos mostrado entonces que

L-\varepsilon<a_{n}\leq L para cada n\geq N.

o equivalentemente

|L-a_{n}|<\varepsilon para cada n\geq N,

la cual es precisamente la definición del límite.

\blacksquare

Notar entonces que en nuestro problema, podemos escribir a_{n} como la suma de las áreas de los primeros n cuadrados. Naturalmente, cada vez sumamos mas área, por lo que esta sucesión es no decreciente. Por último, es claro que la suma total es menor al area del cuadrado mayor, la cual es uno, por lo que a_{n} está acotada por dicha área. Con esto, y gracias a nuestra reciente demostración, hemos probado que sumar las infinitas areas nos da un valor finito.

En la practica sin embargo, este problema es bastante mas fácil si lo abordamos directamente con cálculos numéricos. En efecto, el área de cada cuadrado es \frac{1}{4^{i}}, por lo que la suma de las áreas de los primeros i cuadrados nos queda

\sum_{i=1}^{n}\frac{1}{4^{i}}

y si llamamos a_{n} a tal suma, como antes, obtenemos

\frac{1}{4}a_{n}=\frac{1}{4}\sum_{i=1}^{n}\frac{1}{4^{i}}=\sum_{i=2}^{n+1}\frac{1}{4^{i}}

por lo que

\frac{3}{4}a_{n}=a_{n}-\dfrac{1}{4}a_{n}=\sum_{i=1}^{n}\frac{1}{4^{i}}-\sum_{i=2}^{n+1}\frac{1}{4^{i}}=\frac{1}{4}-\frac{1}{4^{n+1}}

de donde se concluye que

a_{n}=\frac{4}{3}(\frac{1}{4}-\frac{1}{4^{n+1}})

con lo que cuando n tiende a infinito, nos queda L=\frac{1}{3}.

Hemos concluido entonces que el área azul en nustra figura es de \frac{1}{3}. Quizás en la figura siguiente queda un poco mas graficado este resultado.

fig2

Un problema sobre convergencias

Definición: Un conjunto I\subset\mathbb{N} se dice de densidad cero si

\lim_{n\rightarrow\infty}\dfrac{\sharp(I\cap\{0,...,n-1\})}{n}=0.

Si consideramos ahora una sucesión \{a_{n}\}_{n\in\mathbb{N}} de números reales, acotada, nos podemos preguntar si existe un subconjunto I\subset\mathbb{N} de densidad cero tal que el límite

\lim_{n\not\in I}a_{n}

existe.

En el caso particular cuando \lim_{n\not\in I}a_{n}=0 para algún conjunto I se tiene una caracterización bastante interesante.

 

Proposición: Para \{a_{n}\}_{n\in\mathbb{N}} como antes, las afirmaciones siguientes son equivalentes

  1. \lim_{n\rightarrow\infty}\frac{1}{n}\sum_{i=0}^{n-1}|a_{i}|=0
  2. Existe I\subset\mathbb{N} de densidad cero tal que \lim_{n\not\in I}a_{n}=0.

Notación: Denotamos \delta(I)=\lim_{n\rightarrow\infty}\dfrac{\sharp(I\cap\{0,...,n-1\})}{n}, \delta_{n}(I)=\dfrac{\sharp(I\cap\{0,...,n-1\})}{n} y C(I,n)=\sharp(I\cap\{0,...,n-1\}).

 

Para mostrar que (1) implica (2), consideramos un entero k\geq 1 y el conjunto

I_{k}=\{n\geq 0:|a_{n}|\geq\frac{1}{k}\},

entonces dicho conjunto es de densidad cero pues

\frac{1}{n}\sum_{i=0}^{n-1}|a_{i}|\geq\frac{1}{n}\sum_{i=0,i\in I_{k}}^{n-1}|a_{i}|\geq\frac{1}{k}\delta_{n}(I_{k})

y por lo tanto, como el término de la izquierda tiende a cero, el de la derecha también cuando n tiende a infinito.

Hasta ahora tenemos lo siguiente, para cada k\geq 0 y para cada \varepsilon>0 existe N(k,\varepsilon)\in\mathbb{N} tal que si n>N(k,\varepsilon) entonces \delta_{n}(I_{k})<\varepsilon. Por lo tanto, si escogemos \varepsilon=\frac{1}{k}, podemos escoger un entero N(k)>0 tal que \delta_{n}(I_{k})<\frac{1}{k} para n>N(k). Mas aún, podemos escoger los términos N(k) de modo que sean estrictamente crecientes. Escribimos en tal caso \{n_{k}=N(k)\}_{k} una sucesión estrictamente creciente de números naturales con la propiedad siguiente: Para todo n>n_{k} se tiene

\delta_{n}(I_{k})<\frac{1}{k}.

Escribimos I=\bigcup_{k\geq 0}(I_{k+1}\cap\{n_{k},...,n_{k+1}-1\}). Entonces para cada n\in\{n_{k},...,n_{k+1}-1\} se tiene

I\cap\{0,...,n-1\}\subset I_{k+1}\cap\{0,...,n-1\}

y luego al considerar la cardinalidad de tales conjuntos y dividir por n, nos queda

\delta_{n}(I)\leq\delta_{n}(I_{k+1})\leq\frac{1}{k+1}

de donde, haciendo tender k a infinito (y por lo tanto n también), resulta que

\lim_{n\rightarrow\infty}\delta_{n}(I)=0.

Finalmente si n\geq n_{k} no pertenece a I, entonces n no pertenece a I_{k+1} y por lo tanto |a_{n}|<\frac{1}{k+1}, de donde se concluye que \lim_{n\not\in I}a_{n}=0.

Para el recíproco ((2) implica (1)), consideramos K>0 una cota superior de \{|a_{n}|\}_{n\geq 0}. Para cada \varepsilon>0 existe N(\varepsilon) tal que si n>N(\varepsilon) entonces C(I,n)<\varepsilon n y además |a_{n}|<\varepsilon si n\not\in I. Se deduce entonces que para cada n>\max\{N(\varepsilon),\frac{KN(\varepsilon)}{\varepsilon}\}

\frac{1}{n}\sum_{i=0}^{n-1}|a_{i}|=\frac{1}{n}[\sum_{i\in I}^{n-1}|a_{i}|+\sum_{i<N(\varepsilon),i\not\in I}^{n-1}|a_{i}|+\sum_{N(\varepsilon)\leq i<n,i\not\in I}^{n-1}|a_{i}|]

\leq\frac{KC(I,n)}{n}+\frac{KN(\varepsilon)}{n}+\varepsilon\leq(K+2)\varepsilon

lo que concluye la demostración.

Criterio de convergencia: Cuociente.

Cuando en los cursos de cálculo comienzan a enseñar sucesiones, uno de los primeros criterios de convergencia que se estudia es el Criterio del Cuociente. Para los que no lo conocen, este dice lo siguiente:

«Sea (a_{n})_{n\in\mathbb{N}} sucesión de términos positivos. Suponga que \lim_{n\rightarrow\infty}\frac{a_{n+1}}{a_{n}}=L<1, entonces \lim_{n\rightarrow\infty}a_{n}=0«.

Evidentemente existen muchos ejemplos en que este criterio es útil, uno de ellos es el siguiente

Ejemplo: Sea a_{n}=\frac{n!}{n^{n}}. ¿Tendra límite esta sucesión?. Intuitivamente podemos decir que n^{n} crece mas rápido que n! y por lo tanto el límite existe y es cero (¡pero en matemáticas la intuición no es suficiente!). El criterio anterior nos asegura que estamos en lo correcto, y que además, el crecimiento del denominador es lo suficientemente más rápido que el crecimiento del numerador como para asegurar que la sucesion tiende a cero;  en efecto,

\lim_{n\rightarrow\infty}\frac{a_{n+1}}{a_{n}}=\lim_{n\rightarrow\infty}\frac{(n+1)!n^{n}}{(n)!(n+1)^{n+1}}

y luego

\lim_{n\rightarrow\infty}\frac{a_{n+1}}{a_{n}}=\lim_{n\rightarrow\infty}\frac{1}{(1+\frac{1}{n})^n}=\frac{1}{e}.

Y así, como \frac{1}{e}<1 se tiene que \lim_{n\rightarrow\infty}\frac{n!}{n^{n}}=0.

Probaremos entonces un resultado un poco mas general.

Proposición: Sea (a_{n})_{n\in\mathbb{N}} sucesión de términos positivos tal que existe un natural N que satisface

\frac{a_{n+1}}{a_{n}}<K<1

para todo n\geq N. Entonces \lim_{n\rightarrow\infty}a_{n}=0.

Observacion: Es importante darse cuenta que nuestra hipótesis no implica que la sucesion de terminos \frac{a_{n+1}}{a_{n}} sea convergente. 

Consideremos N de nuestra hipótesis. Para n>N definimos m=n-N. Entonces

a_{n}=a_{N+m}<Ka_{N+m-1}<K^{2}a_{N+m-2}<...<K^{m}a_{N}

entonces, de aquí es claro que

0\leq a_{n}<K^{n}\frac{a_{N}}{K^{N}}

para cada n\geq N. Como \frac{a_{N}}{K^{N}} es una constante y 0\leq K<1 (lo que implica que el término de la derecha tiende a 0), concluímos aplicando límite (y Teorema del Sandwich) que

\lim_{n\rightarrow\infty}a_{n}=0.

Observaciones:

  • La proposición implica el criterio del cuociente (usual), pues si el límite del cuociente tiende a cero, entonces para \varepsilon>0 suficientemente pequeño tal que L+\varepsilon<1 (de ahi la importancia que L<1), entonces existe N natural tal que el cuociente \frac{a_{n+1}}{a_{n}} es menor que L+\varepsilon para n\geq N. Solo basta considerar K=L+\varepsilon.
  • Un criterio equivalente al que acabamos de mostrar es el siguiente. Si (a_{n})_{n\in\mathbb{N}} como en la proposición, y \limsup_{n\rightarrow\infty}\frac{a_{n+1}}{a_{n}}\leq K<1, entonces la sucesión tiende a cero. La equivalencia de estos dos resultados no es difícil de ver y es dejada al lector como ejercicio.

Sobre puntos de acumulación y límite superior

Sea (a_{n})_{\mathbb{N}} una sucesión de números reales. Demuestre que x es un punto de acumulación de (a_{n})_{\mathbb{N}} sí y sólo sí (a_{n})_{\mathbb{N}} tiene una subsucesión (a_{\phi(n)})_{\mathbb{N}} tal que \lim_{n\rightarrow\infty}a_{\phi(n)}=x. Demuestre además que el límite superior de una sucesión es el mayor de sus puntos de acumulación.

Recordemos que x es un punto de acumulación sí y sólo sí para todo \varepsilon>0 el conjunto \{k\in\mathbb{N}:|x-a_{k}|<\varepsilon\} es infinito.

Sea entonces \varepsilon_{n}=\frac{1}{n}, luego existe N(n)\in\mathbb{N} tal que |x-a_{N(n)}|<\varepsilon_{n}. Como \{k\in\mathbb{N}:|x-a_{k}|<\varepsilon_{n}\} es infinito, podemos escoger los naturales N(n) de manera estrictamente creciente. Es decir, para \phi:\mathbb{N}\rightarrow\mathbb{N}, definida por n\mapsto N(n), nos queda

\lim_{n\rightarrow\infty}a_{\phi(n)}=\lim_{n\rightarrow\infty}a_{N(n)}=x

por construcción.

Observación: Para cada \varepsilon>0 existe k natural tal que si n>k entonces \frac{1}{n}<\varepsilon. En el caso anterior esto implica que |x-a_{\phi(n)}|<\varepsilon para n>k. De ahí la convergencia de la subsucesión.

Para el recíproco, basta notar que por definición de límite, la existecia de una subsucesión (a_{\phi(n)})_{n\in\mathbb{N}} implica que para cada \varepsilon el conjunto \{k\in\mathbb{N}:|x-a_{k}|<\varepsilon\} es infinito pues en particular todos los naturales k=\phi(n) pertenecen a dicho conjunto para n\geq N\in\mathbb{N} (cuya existencia proviene de la subsucesión convergente).

Para el segundo problema, nuevamente consideramos x punto de acumulación, y sea (a_{\phi(n)}) convergente a x. Recordemos que

\limsup_{n\rightarrow\infty}a_{n}=\lim_{n\rightarrow\infty}\sup\{a_{n},a_{n+1},...\}

Como \phi es función creciente tal que \phi(n)\geq n, se concluye

a_{\phi(n)}\leq\sup\{a_{n},a_{n+1},...\}

Y luego aplicando límite en ambos lados (y como ambas sucesiones son convergentes!) se concluye que x\leq\limsup_{n\rightarrow\infty}a_{n}. Por arbitrariedad de x se tiene que el límite superior es el mayor de los puntos de acumulación.

Sobre la convergencia y convergencia absoluta

Demuestre que si \lim_{n \rightarrow \infty} a_{n}=L, entonces \lim_{n \rightarrow \infty} |a_{n}|=|L|. ¿Es cierto el recíproco?.

Por definición de límite se tiene que para cada \varepsilon >0 existe un natural N tal que para n>N se satisface |a_{n}-L|< \varepsilon. Por otro lado, por desigualdad triangular, se tiene que para cualquier x,y \in \mathbb{R}, se cumple la desigualdad

||x|-|y|| \leq |x-y|

Por lo que ||a_{n}|-|L|| \leq |a_{n}-L| < \varepsilon. Esto prueba que para todo \varepsilon > 0 existe N \in \mathbb{N} tal que

||a_{n}|-|L||< \varepsilon

para n>N. Esto se traduce en \lim_{n \rightarrow \infty} |a_{n}|=|L|

El recíproco es falso, basta considerar la sucesión que vale 1 para los pares y -1 para los impares. La sucesión no converge, pero su valor absoluto es constante e igual a 1, por lo que su límite es 1.